Definite Integral Word Problem

Click For Summary
To approximate the volume of water in a rectangular pool measuring 10 meters wide and 25 meters long, a left-hand Riemann Sum with 5 terms is suggested. The depth of the water varies from 1 meter at the shallow end to 4.57 meters at the deep end. A definite integral can be formulated to calculate the exact volume, but the initial steps involve visualizing the pool's cross-section and determining the heights at specific intervals. The user expresses confusion about the problem and the heights not displaying correctly. Clarification and guidance are sought to better understand the calculations involved.
niravana21
Messages
34
Reaction score
0

Homework Statement


A rectangular pool is 10 meters wide and 25 meters long. The depth of water "x" meters from the shallow end of the pool is
MSP22607039579497235809_347MSPStore.gif


Part A. Write a left-hand Riemann Sum with 5 terms that approximates the volume of the water in the pool. Is your approximation an underestimate or an overestimate? Explain.

Part B. Write a definite integral that gives the exact volume of the water in the pool.

The Attempt at a Solution


Don't know where I should even begin. Forgot my book at school and its vacation time:(
 
Physics news on Phys.org
Draw a picture looking at the pool from the side. It should be a quadrilateral with base 25 m long, two right angle ends, and end heights 0^2/175+ 1= 1 and 25^2/175+ 1= 4.57 meters (not a very deep pool!). A cross section, of width h at distance x from the shallow end, is 10 by 4.57 by h. Take h= 25/n where n is the number of steps you decide to use.
 
i still don't quite understand :(
Your heights aren't showing up.
 
anyone?
 
Question: A clock's minute hand has length 4 and its hour hand has length 3. What is the distance between the tips at the moment when it is increasing most rapidly?(Putnam Exam Question) Answer: Making assumption that both the hands moves at constant angular velocities, the answer is ## \sqrt{7} .## But don't you think this assumption is somewhat doubtful and wrong?

Similar threads

Replies
4
Views
4K
  • · Replies 3 ·
Replies
3
Views
5K
  • · Replies 18 ·
Replies
18
Views
3K
  • · Replies 1 ·
Replies
1
Views
2K
  • · Replies 2 ·
Replies
2
Views
4K
  • · Replies 14 ·
Replies
14
Views
4K
  • · Replies 5 ·
Replies
5
Views
6K
Replies
3
Views
3K
  • · Replies 6 ·
Replies
6
Views
2K
Replies
1
Views
2K